Ahaan's polynomial roots

Algebra Level 2

Suppose u u , v v , and w w are the roots of polynomial f ( x ) = x 3 + 20 x 2 + 13 x + 42 , f(x) = x^3 + 20x^2 + 13x + 42, and 1 u \frac {1}{u} , 1 v \frac {1}{v} , and 1 w \frac {1}{w} are the roots of polynomial g ( x ) = x 3 + r x 2 + s x + t . g(x) = x^3 + rx^2 + sx + t. If g ( 1 ) = a b g(1) = \frac {a}{b} , where a a and b b are coprime positive integers, what is the value of a + b a + b ?

This problem is posed by Ahaan Rungta .


The answer is 59.

This section requires Javascript.
You are seeing this because something didn't load right. We suggest you, (a) try refreshing the page, (b) enabling javascript if it is disabled on your browser and, finally, (c) loading the non-javascript version of this page . We're sorry about the hassle.

15 solutions

Trevor B.
Oct 30, 2013

If the function f ( x ) = a x 3 + b x 2 + c x + d f(x)=ax^3+bx^2+cx+d has roots w w , x x , and y y , then the function with roots 1 w \frac{1}{w} , 1 x \frac{1}{x} , and 1 y \frac{1}{y} can be represented by d x 3 + c x 2 + b x + a dx^3+cx^2+bx+a .

Since we want g ( x ) g(x) to have a leading coefficient of 1 1 , we divide by d = 42 d=42 to get

g ( x ) = x 3 + 13 42 x 2 + 20 42 x + 1 42 g(x) = x^3 + \frac{13}{42} x^2 + \frac{20}{42} x + \frac{1}{42}

Hence, g ( 1 ) = 76 42 = 38 21 g(1) = \frac{76}{42} = \frac{38}{21} , and thus a + b = 59 a+b = 59 .

Moderator note:

I converted this comment into a solution, because it elegantly demonstrates how choosing the correct perspective on a problem could greatly simplify the work that needs to be done. Compare it with other solutions which use Vieta's formula to justify g ( x ) g(x) .

I don't get this solution, is there a proof for it? (switching the coefficients)

Sameer L. - 7 years, 7 months ago

Log in to reply

Consider f ( x ) = 0 f(x)=0 , with w w as a root, thus, f ( w ) = a ( w ) 3 + b ( w ) 2 + c ( w ) + d f(w) = a(w)^3 + b(w)^2 + c(w) + d . Dividing by 1 w 3 \frac{1}{w^3} throughout, you get f ( w ) w 3 = a + b w + c w 2 + d w 3 \frac{f(w)}{w^3} = a + \frac{b}{w} + \frac{c}{w^2} + \frac{d}{w^3} Hence, a + b w + c w 2 + d w 3 a + \frac{b}{w} + \frac{c}{w^2} + \frac{d}{w^3} = a + b ( 1 w ) + c ( 1 w 2 ) + d ( 1 w 3 ) = a + b(\frac{1}{w}) + c(\frac{1}{w^2}) + d(\frac{1}{w^3}) = d ( x ) 3 + c ( x ) 2 + b ( x ) + a = d(x)^3 + c(x)^2 + b(x) + a for an equation with roots 1 w \frac{1}{w}

Tan Kiat - 7 years, 7 months ago

Log in to reply

Great job. Using the substitution w = 1 x w = \frac{1}{x} allows us to transform the roots directly. This works for many other cases, as long as the resultant can be made into a polynomial (of the same degree).

If α i \alpha_i are the roots of f ( x ) f(x) , what polynomial g ( x ) g(x) has roots 2 α i + 3 2 \alpha_i + 3 ?

Calvin Lin Staff - 7 years, 7 months ago

Log in to reply

@Calvin Lin Is it 2 f ( x ) + 3 2 f(x) + 3 ?

Sameer L. - 7 years, 7 months ago

Log in to reply

@Sameer L. Can you test your guess?

Does it work for the polynomial f ( x ) = x f(x) = x which has a root of 0 0 ?

Does it work for the polynomial f ( x ) = x ( x 1 ) f(x) = x(x-1) ?

Calvin Lin Staff - 7 years, 7 months ago

Log in to reply

@Calvin Lin Calvin Lin is it we have to substitute (x-3)/2 in place of x in f(x) to get g(x) ??

RAJ RAJPUT - 5 years, 7 months ago

May I know how do you prove it? Thanks.

Wee Hau Chin - 7 years, 7 months ago

Wow!

Rindell Mabunga - 7 years, 7 months ago

You made a coment on Noor muhammad's solution 3 days, 12 hours ago, and you post this solution 1 day 22 hours ago right now. I think we are not allowed to do that

Jordi Bosch - 7 years, 7 months ago

Log in to reply

Actually, I converted his comment into a solution, to illustrate how having the correct perspective on a problem could simplify it greatly.

My apologies, I'd add in an explanation.

Calvin Lin Staff - 7 years, 7 months ago

This is very elegant! However, although we barely use the actual roots in this solution, I do dislike having x x as a root and a variable. I myself did not see this solution, and instead went the well-trodden path of Vieta's.

Nathan Weckwerth - 7 years, 7 months ago

Elegant solution for this problem! When I submitted it, I used the Vieta's solution. Nice job. Upvoted. Out of curiosity, which level was this problem in?

Ahaan Rungta - 7 years, 7 months ago

Log in to reply

It was level 4 problem 2

Jordi Bosch - 7 years, 7 months ago

Log in to reply

Makes sense.

Ahaan Rungta - 7 years, 6 months ago

Given f ( x ) = x 3 + 20 x 2 + 13 x + 42 f(x) = x^{3} + 20x^{2} + 13x + 42
Let u, v, w be the roots of the polynomial, then according to Vieta's Formula,
u + v + w = 20 u+v+w = -20
u v + u w + v w = 13 uv+uw+vw = 13
u v w = 42 uvw = -42
Now, the required polynomial is g ( x ) = x 3 + r x 2 + s x + t g(x) = x^{3} + rx^{2} +sx + t with roots 1 u 1 v 1 w \frac{1}{u} \frac{1}{v} \frac{1}{w}
Applying Vieta's Formula on this polynomial we have:
1 u + 1 v + 1 w = r \frac{1}{u} + \frac{1}{v} + \frac{1}{w} = -r
v w + u w + u v u v w = r \frac{vw+uw+uv}{uvw} = -r
Using values of expressions;
13 42 = 13 42 \frac{13}{-42} = \frac{-13}{42} = -r
r = 13 42 r= \frac{13}{42}
Also for t,
1 v . 1 u . 1 w = t \frac{1}{v}.\frac{1}{u}.\frac{1}{w} = -t 1 u v w = t \frac{1}{uvw} = -t Substituting value for uvw we have;
1 42 = t \frac{1}{-42} = -t
t = 1 42 t= \frac{1}{42}
Also for s,
1 u v + 1 u w + 1 v w = s \frac{1}{uv} + \frac{1}{uw} + \frac{1}{vw} = s
w + v + u u v w = s \frac{w+v+u}{uvw} = s
Substituting values for u+v+w and uvw we have;
s = 20 42 s = \frac{-20}{-42}
s = 20 42 s = \frac{20}{42}
Now,
By substituting the values for r, s and t in polynomial g(x) we have;
g ( x ) = x 3 + 13 42 x 2 + 20 42 x + 1 42 g(x) = x^{3} +\frac{13}{42}x^{2} + \frac{20}{42}x + \frac{1}{42}
Therefore,
g ( 1 ) = 1 + 13 42 + 20 42 + 1 42 g(1) = 1 + \frac{13}{42} + \frac{20}{42} +\frac{1}{42}
g ( 1 ) = 42 + 13 + 20 + 1 42 = 76 42 = 38 21 g(1) = \frac{42+13+20+1}{42} = \frac{76}{42} = \frac{38}{21}
= > a = 38 , b = 21 => a= 38, b=21
a + b = 38 + 21 a+b = 38+21
a + b = 59 A n s w e r \boxed{a+b = 59 Answer}



It seems like everyone approached this using Vieta's. There isn't a need to do so. How does g ( x ) g(x) relate to f ( x ) f(x) ? Do you see a quick way to go from f ( x ) f(x) to g ( x ) g(x) ?

Calvin Lin Staff - 7 years, 7 months ago

Log in to reply

If the function f ( x ) = a x 3 + b x 2 + c x + d f(x)=ax^3+bx^2+cx+d has roots w w , x x , and y y , then the function g ( x ) g(x) with roots 1 w \frac{1}{w} , 1 x \frac{1}{x} , and 1 y \frac{1}{y} can be represented by g ( x ) = d x 3 + c x 2 + b x + a g(x)=dx^3+cx^2+bx+a .

Trevor B. - 7 years, 7 months ago

Log in to reply

Perfect! This allows you to skip all of the steps shown in these solutions.

Calvin Lin Staff - 7 years, 7 months ago
Rindell Mabunga
Oct 28, 2013

By Vieta's Theorem in f ( x ) f(x) yields:

  • u + v + w = 20 u + v + w = -20
  • u v + v w + w u = 13 uv + vw + wu = 13
  • u v w = 42 uvw = -42

Using Vieta's Theorem in g ( x ) g(x) also yields;

  • 1 u + 1 v + 1 w = r \frac{1}{u} + \frac{1}{v} + \frac{1}{w} = -r
  • 1 u v + 1 v w + 1 w u = s \frac{1}{uv} + \frac{1}{vw} + \frac{1}{wu} = s
  • 1 u v w = t \frac{1}{uvw} = -t

We must solve for r, s, and t to determine the value of g ( 1 ) g(1)

First solving for r,

1 u + 1 v + 1 w = u v + v w + w u u v w = 13 42 = 13 42 = r \frac{1}{u} + \frac{1}{v} + \frac{1}{w} = \frac{uv + vw + wu}{uvw} = \frac{13}{-42} = \frac{13}{-42} = -r

Therefore, r = 13 42 r = \frac{13}{42}

Solving for s,

1 u v + 1 v w + 1 w u = u + v + w u v w = 20 42 = 20 42 = s \frac{1}{uv} + \frac{1}{vw} + \frac{1}{wu} = \frac{u + v + w}{uvw} = \frac{-20}{-42} = \frac{20}{42} = s

Lastly, solving for t, 1 u v w = 1 42 = t \frac{1}{uvw} = \frac{1}{-42} = -t

Therefore, t = 1 42 t = \frac{1}{42}

We are now able to solve for g ( 1 ) g(1) ,

g ( 1 ) = 1 3 + r ( 1 2 ) + s ( 1 ) + t = 1 + r + s + t = 1 + 13 42 + 20 42 + 1 42 = 76 42 = 38 21 g(1) = 1^{3} + r ( 1^{2} ) + s ( 1 ) + t = 1 + r + s + t = 1 + \frac{13}{42} + \frac{20}{42} + \frac{1}{42} = \frac{76}{42} = \frac{38}{21}

Therefore

a = 38 , b = 21 a = 38 , b = 21

and

a + b = a + b = 59

It seems like everyone approached this using Vieta's. There isn't a need to do so. How does g ( x ) g(x) relate to f ( x ) f(x) ? Do you see a quick way to go from f ( x ) f(x) to g ( x ) g(x) ?

Calvin Lin Staff - 7 years, 7 months ago
Andreas Asheim
Sep 2, 2016

Start from the factorisation of the two polynomials:

f ( x ) = ( x u ) ( x v ) ( x w ) f(x)=(x-u)(x-v)(x-w)

g ( x ) = ( x 1 u ) ( x 1 v ) ( x 1 w ) g(x)=(x-\frac 1 u)(x-\frac 1 v)(x-\frac 1 w)

From there, just multiply g ( 1 ) g(1) by f ( 0 ) = u v w f(0)=uvw

g ( 1 ) = ( 1 1 u ) ( 1 1 v ) ( 1 1 w ) g(1)=(1-\frac 1 u)(1-\frac 1 v)(1-\frac 1 w)

u v w g ( 1 ) = ( 1 u ) ( 1 v ) ( 1 w ) uvw g(1)=-(1-u)(1-v)(1-w)

f ( 0 ) g ( 1 ) = f ( 1 ) -f(0)g(1)=-f(1)

g ( 1 ) = 76 42 = 38 21 g(1) = \frac{76}{42}=\frac{38}{21}

The solution is therefore 38 + 21 = 59 38+21=59

Because 1 u \frac {1} {u} is a root of g ( x ) g(x) , we get:

g ( 1 u ) = 1 u 3 + 1 u 2 + s u + t = 0 g(\frac {1} {u} ) = \frac {1} {u^3} + \frac {1} {u^2} + \frac {s} {u} + t = 0

Multiplying by 42 u 3 42 u^3 we get:

42 t u 3 + 42 s u 2 + 42 r u + 42 = 0 42tu^3 + 42su^2 + 42ru + 42 = 0

Because f ( u ) = u 3 + 20 u 2 + 13 u + 42 = 0 f(u) = u^3 + 20u^2 + 13u + 42 = 0 , the unknown coefficients can be determined by:

42 t = 1 42 s = 20 42 r = 13 42t=1\\ 42s=20\\ 42r=13

t = 1 42 s = 20 42 r = 13 42 t=\frac {1} {42} \\ s = \frac {20} {42} \\ r = \frac {13} {42}

g ( 1 ) = 1 + r + s + t = 1 + 13 42 + 20 42 + 1 42 = 38 21 g(1) = 1 + r + s + t = 1 + \frac {13} {42} + \frac {20} {42} + \frac {1} {42} = \frac {38} {21}

a + b = 38 + 21 = 59 a + b = 38 + 21 = \boxed {59}

William Isoroku
Dec 31, 2014

By Vieta's formulas, we can conclude that u + v + w = 20 u+v+w=-20 , u v + u w + v w = 13 uv+uw+vw=13 and u v w = 42 uvw=-42 .

By Vieta's again for g ( x ) g(x) :

1 u + 1 v + 1 w = u v + u w + v w u v w 13 42 \frac { 1 }{ u } +\frac { 1 }{ v } +\frac { 1 }{ w } =\frac { uv+uw+vw }{ uvw } \Longrightarrow \frac { 13 }{ -42 } r = 13 42 \therefore r=\frac { 13 }{ 42 }

1 u v + 1 u w + 1 v w = u + v + w u v w 20 42 \frac { 1 }{ uv } +\frac { 1 }{ uw } +\frac { 1 }{ vw } =\frac { u+v+w }{ uvw } \Longrightarrow \frac { 20 }{ 42 } s = 20 42 \therefore s=\frac { 20 }{ 42 }

1 u v w = 1 42 \frac { 1 }{ uvw } =\frac { 1 }{ -42 } t = 1 42 \therefore t=\frac { 1 }{ 42 }

So, g ( x ) = x 3 + 13 42 x 2 + 20 42 x + 1 42 g(x)= { x }^{ 3 }+\frac { 13 }{ 42 } { x }^{ 2 }+\frac { 20 }{ 42 } x+\frac { 1 }{ 42 }

When x = 1 , g ( x ) = 38 21 = a b x=1, g(x)=\frac{38}{21}=\frac{a}{b} , so a + b = 59 a+b=\boxed{59}

Saleh Elsbahi
Feb 24, 2014

put 1/x at f(x)

we get 1/x^3 + 20/x^2 + 13/x +1

  • x^3

42x^3 + 13x^2 + 20x +1

divided /42

g(x) = x^3 + 13/42 x^2 + 20/42 x + 1/42

g(1) = 38/21

a+b =59

Abubakarr Yillah
Jan 18, 2014

f ( x ) = x 3 + 20 x 2 + 13 x + 42 f({x})={x^3+20x^2+13x+42} So a = 1 , b = 20 , c = 13 , d = 42 {a=1}, {b=20}, {c=13}, {d=42} u + v + w = b a = 20 {u+v+w}=\frac{-b}{a}={-20} u v w = d a = 42 {uvw}=\frac{-d}{a}={-42} u v + u w + v w = c a = 13 {uv+uw+vw}=\frac{c}{a}={13} w v u v w + u w u v w + u v u v w = 1 u + 1 v + 1 w = 13 42 {\frac{wv}{uvw}+\frac{uw}{uvw}+\frac{uv}{uvw}=\frac{1}{u}+\frac{1}{v}+\frac{1}{w}=\frac{-13}{42}} 1 u v w = 1 42 {\frac{1}{uvw}=\frac{-1}{42}} w u v w + v u v w + u u v w = 1 u v + 1 u w + 1 v w = 20 42 = 10 21 {\frac{w}{uvw}+\frac{v}{uvw}+\frac{u}{uvw}=\frac{1}{uv}+\frac{1}{uw}+\frac{1}{vw}=\frac{-20}{-42}=\frac{10}{21}} g ( x ) = x 3 + r x 2 + s x + t g({x})={x^3+rx^2+sx+t} So a = 1 , b = r , c = s , d = t {a=1}, {b=r}, {c=s}, {d=t} 1 u + 1 v + 1 w = b a = r = 13 42 {\frac{1}{u}+\frac{1}{v}+\frac{1}{w}=\frac{-b}{a}={-r}=\frac{-13}{42}} from which r = 13 42 {r}=\frac{13}{42} 1 u v w = d a = t = 1 42 {\frac{1}{uvw}=\frac{-d}{a}={-t}=\frac{-1}{42}} from which t = 142 {t}={1}{42} 1 u v + 1 u w + 1 v w = c a = s = 10 21 {\frac{1}{uv}+\frac{1}{uw}+\frac{1}{vw}=\frac{c}{a}={s}=\frac{10}{21}} from which s = 10 21 {s}=\frac{10}{21} thus g ( x ) = x 3 + 13 42 x 2 + 10 21 x + 1 42 g({x})={x^3+\frac{13}{42}x^2+\frac{10}{21}x+\frac{1}{42}} and g ( 1 ) = 1 + 13 42 + 10 21 + 1 42 g({1})={{1}+\frac{13}{42}+\frac{10}{21}+\frac{1}{42}} which equals g ( 1 ) = 38 21 g({1})=\frac{38}{21} but g ( 1 ) = a b = 38 21 g({1})=\frac{a}{b}=\frac{38}{21} Hence a + b = 38 + 21 = 59 {a+b}={38+21}=\boxed{59}

Ryan Soedjak
Nov 16, 2013

I will generalize the problem:

Suppose u u , v v , w w are the roots of polynomial f ( x ) = x 3 + a x 2 + b x + c , f(x)=x^3+ax^2+bx+c, where c 0 c\not=0 and 1 u \dfrac1u , 1 v \dfrac1v , 1 w \dfrac1w are the roots of polynomial g ( x ) = x 3 + r x 2 + s x + t g(x)=x^3+rx^2+sx+t Find the coefficients of g ( x ) g(x) in terms of a , b , c a,b,c .

By Vieta on f f , we know that u + v + w = a , (eqn. 1) u v + u w + v w = b , (eqn. 2) u v w = c . (eqn. 3) \begin{aligned} u+v+w&=-a,&\text{ (eqn. 1)}\\ uv+uw+vw&=b,&\text{ (eqn. 2)}\\ uvw&=-c.&\text{ (eqn. 3)} \end{aligned}

By Vieta on g g , we know that r = 1 u + 1 v + 1 w = u v + u w + v w u v w , (eqn. A) s = 1 u v + 1 u w + 1 v w = u + v + w u v w , (eqn. B) t = 1 u v w . (eqn. C) \begin{aligned} -r&=\frac1u+\frac1v+\frac1w=\frac{uv+uw+vw}{uvw},&\text{ (eqn. A)}\\ s&=\frac1{uv}+\frac1{uw}+\frac1{vw}=\frac{u+v+w}{uvw},&\text{ (eqn. B)}\\ -t&=\frac1{uvw}.&\text{ (eqn. C)} \end{aligned} Substituting ( eqn. 2 ) (\text{eqn. 2}) and ( eqn. 3 ) (\text{eqn. 3}) into ( eqn. A ) (\text{eqn. A}) gives r = u v + u w + v w u v w = b c = b c . r=-\frac{uv+uw+vw}{uvw}=-\frac{b}{-c}=\frac{b}c. Substituting ( eqn. 1 ) (\text{eqn. 1}) and ( eqn. 3 ) (\text{eqn. 3}) into ( eqn. B ) (\text{eqn. B}) gives s = u + v + w u v w = a c = a c . s=\frac{u+v+w}{uvw}=\frac{-a}{-c}=\frac{a}c. Substituting ( eqn. 3 ) (\text{eqn. 3}) into ( eqn. C ) (\text{eqn. C}) gives t = 1 u v w = 1 c = 1 c . t=-\frac1{uvw}=-\frac1{-c}=\frac1c. Therefore, g ( x ) = x 3 + ( b c ) x 2 + ( a c ) x + 1 c . g(x)=x^3+\left(\frac{b}c\right)x^2+\left(\frac{a}c\right)x+\frac1c.

In this problem, we have g ( 1 ) = 1 + r + s + t = 1 + b c + a c + 1 c = 1 + 13 42 + 20 42 + 1 42 = 38 21 \begin{aligned} g(1)&=1+r+s+t\\ &=1+\frac{b}c+\frac{a}c+\frac1c\\ &=1+\frac{13}{42}+\frac{20}{42}+\frac1{42}\\ &=\frac{38}{21}\end{aligned} as our answer.

Motivation: Well, this is a pretty basic Vieta's formulas exercise, you know stuff about the roots and coefficients.

SHOOT I'M DUMB.

Ryan Soedjak - 7 years, 6 months ago
Wee Hau Chin
Nov 1, 2013

According to Vieta's Formula, u + v + w = 20 1 = 20 u+v+w = -\frac{20}{1} \ = -20 , u v + u w + v w = 13 1 = 13 uv+uw+vw = \frac{13}{1} \ = 13 , u v w = 42 1 = 42 uvw = -\frac{42}{1} \ = -42 . Therefore, r 1 = 1 u + 1 v + 1 w = u v + u w + v w u v w = 13 42 -\frac{r}{1}\ = \frac{1}{u}\ + \frac{1}{v}\ + \frac{1}{w}\ = \frac{uv+uw+vw}{uvw}\ = \frac{13}{-42}\ , r = 13 42 r = \frac{13}{42}\ s 1 = 1 u v + 1 u w + 1 v w = u + v + w u v w = 20 42 \frac{s}{1} = \frac{1}{uv}\ + \frac{1}{uw}\ + \frac{1}{vw}\ = \frac{u+v+w}{uvw} = \frac{-20}{-42}\ , s = 10 21 s = \frac{10}{21}\ . t 1 = 1 u v w = 1 42 -\frac{t}{1}\ = \frac{1}{uvw}\ = \frac{1}{-42}\ , t = 1 42 t = \frac{1}{42}\ . Thus, g ( 1 ) = 1 3 + 13 42 ( 1 2 ) + 10 21 ( 1 ) + 1 42 = 38 21 g(1) = 1^3 + \frac{13}{42}\ (1^2) + \frac{10}{21}\ (1) + \frac{1}{42}\ = \frac{38}{21}\ , a + b = 38 + 21 = 59 a+b = 38+21 = \boxed{59}

Jack Olive
Oct 31, 2013

From the polynomial, we say that
-20 = u + v + w
13 = uv + vw + wu
-42 = uvw
And...
-r = 1 u \frac{1}{u} + 1 v \frac{1}{v} + 1 w \frac{1}{w}
= u v + v w + w u u v w \frac{uv + vw + wu}{uvw} = 13 42 \frac{13}{-42}
r = 13 42 \frac{13}{42}
s = 1 u v \frac{1}{uv} + 1 v w \frac{1}{vw} + 1 w u \frac{1}{wu}
= u + v + w u v w \frac{u + v + w}{uvw} = 20 42 \frac{-20}{-42} = 10 21 \frac{10}{21}
s = 10 21 \frac{10}{21}
-t = 1 u v w \frac{1}{uvw} = 1 42 \frac{1}{-42}
t = 1 42 \frac{1}{42}
Therefore, g(x) = x 3 x^{3} + 13 42 \frac{13}{42} x 2 x^{2} + 10 21 \frac{10}{21} x + 1 42 \frac{1}{42}
then, g(1) = 1 + 13 42 \frac{13}{42} + 10 21 \frac{10}{21} + 1 42 \frac{1}{42} = 38 21 \frac{38}{21} = a b \frac{a}{b}
Thus, a + b = 59 \boxed{59}








Akbar Gumbira
Oct 28, 2013

Since u , v , w u, v, w are the roots of f ( x ) f(x) then:

  1. ( x u ) ( x v ) ( x w ) = 0 (x-u)(x-v)(x-w)=0 or x 3 ( u + v + w ) x 2 + ( u v + u w + v w ) x u v w = 0 x^3-(u+v+w)x^2+(uv+uw+vw)x-uvw=0

By substituting 1 1 to the right coeffiecient of f ( x ) f(x) we can get:

a. u + v + w = 20 u+v+w=-20

b. u v + v w + u w = 13 uv+vw+uw=13

c. u v w = 42 uvw=42

Using the same method for g ( x ) g(x) , we can derive:

  1. 1 u + 1 v + 1 w = r \frac{1}{u}+\frac{1}{v}+\frac{1}{w}=-r

  2. 1 u v + 1 v w + 1 u w = s \frac{1}{uv}+\frac{1}{vw}+\frac{1}{uw}=s

  3. 1 u v w = t \frac{1}{uvw}=-t

By solving 1 , 2 , 3 1, 2, 3 using a , b , c a, b, c we can get:

  1. r = u v + u w + v w u v w = 13 42 r = - \frac{uv+uw+vw}{uvw} = \frac{13}{42}

  2. s = u + v + w u v w = 20 42 s = \frac{u+v+w}{uvw} = \frac{20}{42}

  3. t = 1 u v w = 1 42 t =- \frac{1}{uvw} = \frac{1}{42}

Then g ( 1 ) = 1 + 13 42 + 20 42 + 1 42 = 76 42 = 38 21 g(1) = 1 + \frac{13}{42} + \frac{20}{42} + \frac{1}{42} = \frac{76}{42} = \frac{38}{21}

Since 38 38 and 21 21 are coprime positive integers, then a + b = 59 a+b =\boxed{59}

Correction : u v w = 42 uvw=-42

Akbar Gumbira - 7 years, 7 months ago

I do not understand what "By substituting 1 to the right coefficient of f ( x ) f(x) " means.

The statements follow simply by comparing coefficients, which likely is what you mean.

Calvin Lin Staff - 7 years, 7 months ago
Jordi Bosch
Oct 28, 2013

From Vieta's Formuals we know that:

u + v + w = b a = 20 u + v + w = \frac{-b}{a} =-20

u v + v w + u w = c a = 13 uv + vw + uw = \frac{c}{a} = 13

w u v = d a = 42 wuv = \frac{-d}{a} = -42

Using the same ideas for g ( x ) g(x) :

1 u + 1 v + 1 w = v u + u w + u v u v w = 13 42 = r r = 13 42 \frac{1}{u}+\frac{1}{v} + \frac{1}{w} = \frac{vu+uw+uv}{uvw} = -\frac{13}{42} = -r \rightarrow r = \frac{13}{42}

( 1 u 1 v ) + ( 1 w 1 u ) + ( 1 w 1 v ) = v + u + w w u v = 20 42 = s s = 20 42 (\frac{1}{u}*\frac{1}{v})+(\frac{1}{w}*\frac{1}{u}) + (\frac{1}{w}*\frac{1}{v}) = \frac{v+u+w}{wuv} = \frac{20}{42} = s \rightarrow s = \frac{20}{42}

1 u 1 v 1 w = 1 u v w = 1 42 = d d = 1 42 \frac{1}{u}*\frac{1}{v}*\frac{1}{w}=\frac{1}{uvw}=-\frac{1}{42} = -d \rightarrow d = \frac{1}{42}

g ( x ) = x 3 + 13 42 x 2 + 20 42 x + 1 42 g(x) = x^{3} + \frac{13}{42}x^{2} + \frac{20}{42}x + \frac{1}{42}

g ( 1 ) = 1 + 34 42 = 76 42 = 38 21 38 + 21 = 59 g(1) = 1 + \frac{34}{42} = \frac{76}{42} = \frac{38}{21}\Longrightarrow 38 + 21 = \boxed{59}

It seems like everyone approached this using Vieta's. There isn't a need to do so. How does g ( x ) g(x) relate to f ( x ) f(x) ? Do you see a quick way to go from f ( x ) f(x) to g ( x ) g(x) ?

Calvin Lin Staff - 7 years, 7 months ago
Bhargav Das
Oct 28, 2013

Using Vi e e ta's relations,we get,

For f ( x ) f(x) , u + v + w = 20 u+v+w=-20 , u v + u w + v w = 13 uv+uw+vw=13 & u v w = 42. uvw=-42.

For g ( x ) g(x) , 1 u + 1 v + 1 w = r \frac{1}{u}+\frac{1}{v}+\frac{1}{w}=-r = u v + v w + u w u v w = \frac{uv+vw+uw}{uvw} = > 13 42 = r . => \frac{13}{42}=r.

and 1 u v + 1 u w + 1 v w = s \frac{1}{uv}+\frac{1}{uw}+\frac{1}{vw}=s = u + v + w u v w = > 20 / 42 = s . =\frac{u+v+w}{uvw}=>20/42=s.

Also, 1 u v w = t = > t = 1 42 . \frac{1}{uvw}=-t=>t=\frac{1}{42}.

Therefore, g ( 1 ) = 1 + r + s + t g(1)=1+r+s+t = 1 + 13 42 + 20 42 + 1 42 =1+\frac{13}{42}+\frac{20}{42}+\frac{1}{42} = 38 21 = \frac{38}{21} , which gives required answer as: 38 + 21 = 38+21= 59 \boxed{59}

Ashiqul Islam
Oct 27, 2013

From "de vieta's formula", u + v+ w = - 20, uv + vw + wu = 13, uvw=-42 And again \frac {1}{u} + \frac {1}{v} + \frac {1}{w} = - r \Rightarrow \frac {uv + vw + wu}{uvw} = -r \Rightarrow r= \frac {13}{42}

\frac {1}{uv} + \frac{1}{vw} + \frac{1}{wu} = s \Rightarrow \frac {u + v + w}{uvw}=s \Rightarrow s = \frac {10}{21}

\frac {1}{uvw} = - t \Rightarrow r = \frac {1}{42}

So g(1) = 1 + r + s + t =1 + \frac {13}{42} + \frac {10}{21} + \frac {1}{42} = \frac {38}{21} Our ans is \boxed{59}

0 pending reports

×

Problem Loading...

Note Loading...

Set Loading...